Weerstand integraal

Moderator: physicalattraction

Reageer
Berichten: 27

Weerstand integraal

Beste,

Er werd ons gevraagd de elektrische weerstand te bepalen over een cirkelboog als de vierkantsweerstand gekend is.

Afbeelding

Voorlopig heb ik het volgende:

Afbeelding

Ik kies de breedte en de hoek α infinitesimaal klein om dan tot de volgende uitdrukking te komen:

(hier ben ik niet zeker over)
\(b= dx \)
\(L= d \alpha*x\)
\(R = \frac{R<sub>vierkant</sub>*d \alpha x}{dx}\)
Nu weten we dat deze oneindig kleine weerstandjes parallel moeten geschakeld zijn dus krijgen we:
\(\frac {1}{R} = \frac{dx}{R<sub>vierkant</sub>*d \alphax}\)
al we nu integreren naar x over het rechter lid:

Afbeelding

de uitkomst is dan:

Afbeelding

Nu weet ik niet wat ik verder nog moet doen met die d\alpha als mijn redenering al juist is.

Alvast bedankt

p.s. mijn latex constructies blijken niet te werken -_-....

Gebruikersavatar
Berichten: 7.390

Re: Weerstand integraal

Ik heb je Latex aangepast. Je moet je Latex tikken in het pop-upvenstertje dat je krijgt na in het menuutje links op Latex te klikken, of door je code tussen tex-tags te zetten.
"C++ : Where friends have access to your private members." Gavin Russell Baker.

Gebruikersavatar
Pluimdrager
Berichten: 6.594

Re: Weerstand integraal

Ik moet eerlijk bekennen dat ik er niet veel van begrijp.

Is de dwarsdoorsnede van die circelboog een rechthoek A=b.d

Wat bedoel je met de""Vierkantsweerstand""?

Berichten: 27

Re: Weerstand integraal

Vierkantstweerstand : http://nl.wikipedia.org/wiki/Vierkantsweerstand

Als men in een heel dunne constante dikte werkt is de vierkantsweerstand een soort vervanger van de resistiviteit van een stof. Door elk vierkant oppervlak hoe groot of hoe klein ook is de weerstand dan hetzelfde.

Om de context wat te schetsen, we kregen weerstandspapier met daarop twee zilvercontactjes (1) en (2) op de foto.

De vraag was hoe we de weerstand van een zo een "cirkelboog" met breedte b en met een afstand r tot het middelpunt.

Omdat dit oppervlak gekromd is moet je dus een integraal gebruiken om dat te bereken. Het is bij die berekening dat ik niet goed weet hoe ik het moet aanpakken. Hopelijk maakt dat het wat duidelijker =)

Gebruikersavatar
Pluimdrager
Berichten: 6.594

Re: Weerstand integraal

Maak gebruik van de volgende elementaire wet van Ohm.
\(\vec{j}=\sigma \cdot \vec{E} \)
Daar de vector van de stroomdichtheid en de vector van de elektrische veldsterkte E in dezelfde richting wijzen , mogen we de wet ook zo schrijven.
\(j=\sigma \cdot E \)
We nemen aan dat er een constant spanningsverschil bestaat tussen 1 en 2 . Dit spanningsverschil noemen we V
\(j=\sigma \cdot \frac{V}{L} \)
\(L=\alpha \cdot x \)
\(j=\frac{ \sigma \cdot V }{\alpha \cdot x} \)
\(di=j \cdot dA \)
\(di=j \cdot dx \cdot d \)
\(di=\frac{\sigma \cdot V}{\alpha \cdot x } \cdot dx \cdot d \)
\(i=\frac{\sigma \cdot V \cdot d}{\alpha} \int_{r}^{r+b} \frac{dx}{x} \)
\(i=\frac{\sigma \cdot V \cdot d}{ \alpha} \cdot \ln \frac{r+b}{r} \)
\(R=\frac{V}{i} \)

Berichten: 27

Re: Weerstand integraal

aadkr bedankt voor je reactie.

Ik heb gisterenavond zelf nog eens gezocht en m.b.v. een boek ben ik tot de correcte oplossing gekomen( die kon ik namelijk vergelijken met mijn andere meetresultaten). Het is nogal wat werk om het volledig on latex om te zetten dus heb ik de oplossing die ik heb uit mijn verslag gekopieerd en in onderstaand bestand gestoken.

oplossing

Nogmaals bedankt voor de respons

Reageer